2808NRS Human Pathophysiology and Pharmacology 2 (Module 1)

Description

Module 1 EoT Exam Revision
Kathleen Jackson
Quiz by Kathleen Jackson, updated more than 1 year ago
Kathleen Jackson
Created by Kathleen Jackson over 5 years ago
1488
6

Resource summary

Question 1

Question
A 56-year-old male is diagnosed with coronary heart disease. Which of the following modifiable risk factors would most likely influence development of this disease?
Answer
  • cigarette smoking.
  • his age.
  • the geographic location of home.
  • his sex.

Question 2

Question
In general, atherosclerosis is triggered by:
Answer
  • injured neutrophils.
  • deposited adipose cells.
  • macrophages that engulf LDL.
  • lipid-laden mast cells.

Question 3

Question
Plaques consisting of lipids, cells, fibrin, and debris, often with attached thrombi that can obstruct arteries and arterioles, are referred to as:
Answer
  • Atheromas
  • Sarcomas
  • Fibromas
  • Prothrombi

Question 4

Question
The most common cause of myocardial ischaemia is:
Answer
  • venous emboli.
  • atherosclerosis.
  • vasospasm.
  • arterial emboli from heart valve vegetation.

Question 5

Question
A 32-year-old female presents with lower leg pain and redness. An ultrasound reveals venous thrombus. Which of the following would be most likely to have caused her condition?
Answer
  • atherosclerosis.
  • heart valve damage.
  • bacterial infection.
  • vascular injury and inflammation.

Question 6

Question
A 75-year-old obese female presents to her GP complaining of oedema in the lower extremities. Physical exam reveals that she has varicose veins. Which of the following is a most likely cause?
Answer
  • ischaemia.
  • extreme exercise.
  • long periods of standing.
  • dyslipidemia.

Question 7

Question
A 28-year-old female presents with severe chest pain and shortness of breath. She is diagnosed with pulmonary embolism, which most likely originated from the:
Answer
  • deep veins of the leg.
  • left ventricle.
  • systemic arteries.
  • superficial veins of the body.

Question 8

Question
A 52-year-old male presents with pooling of blood in the veins of the lower extremities and oedema. He is predisposed to:
Answer
  • deep vein thrombosis.
  • skin hyperpigmentation.
  • stasis ulcers.
  • all answers are correct.

Question 9

Question
A 65-year-old female presents to the ED complaining of difficulty swallowing and shortness of breath. A CT scan would most likely reveal an aneurysm in the:
Answer
  • cerebral vessels.
  • illiac arteries.
  • inferior vena cava.
  • thoracic aorta.

Question 10

Question
The build-up of fatty plaques within the wall of arteries is:
Answer
  • Ischaemic heart disease
  • Peripheral vascular disease
  • Atherosclerosis
  • Hypertension

Question 11

Question
Which of the following types of drugs is NOT typically used to treat hypertension?
Answer
  • Thiazide diuretics
  • Nitrates
  • ACE inhibitors
  • Calcium channel blockers

Question 12

Question
Hypertension that is idiopathic is known as:
Answer
  • Angina
  • Secondary
  • Essential
  • Tertiary

Question 13

Question
Risk factors for atherosclerosis include all the following except:
Answer
  • Low levels of LDL blood serum
  • Hypertension
  • Smoking
  • Advanced age

Question 14

Question
A stationary blood clot within an artery is a:
Answer
  • Myocardial infarction
  • Embolus
  • Stroke
  • Thrombus

Question 15

Question
A 62-year-old male presents to his GP complaining of chest pain at rest and on exertion. He does not have a history of coronary artery disease and reports that the pain often occurs at night. He is most likely experiencing which type of angina?
Answer
  • Exertional
  • Stable
  • Variant
  • Unstable

Question 16

Question
A 53-year-old male presents with recurrent chest pain on exertion. He is diagnosed with angina pectoris. The pain he experiences occurs when:
Answer
  • the vagus nerve is stimulated.
  • myocardial stretch has exceeded the upper limits.
  • the myocardial oxygen supply has fallen below demand.
  • cardiac output has fallen below normal levels.

Question 17

Question
A 51-year-old male is at his health clinic for an annual physical exam. After walking from the car to the clinic, he developed a substernal pain. He also reported discomfort in his left shoulder and his jaw, lasting two to three minutes and then subsiding with rest. He indicates that this has occurred frequently over the past few months with similar exertion. He is most likely to be experiencing:
Answer
  • stable angina
  • unstable angina
  • variant angina
  • myocardial infarction

Question 18

Question
A 52-year-old female is diagnosed with coronary heart disease. She would be expected primarily to suffer from myocardial:
Answer
  • hypertrophy.
  • ischaemia.
  • necrosis.
  • dilation.

Question 19

Question
Cardiac cells can withstand ischaemic conditions for _____ before irreversible cell injury occurs.
Answer
  • 30 minutes
  • 60 minutes
  • 1 minute
  • 20 minutes

Question 20

Question
A 49-year-old male presents to his GP complaining of chest pain. An ECG reveals ST elevation. He is diagnosed with myocardial ischaemia. Which of the following interventions would be most beneficial?
Answer
  • decrease in ventricular volume with diuretic
  • increase cardiac contractility
  • increase heart rate
  • increase myocardial oxygen supply

Question 21

Question
A 55-year-old male died of myocardial infarction. Autopsy would most likely reveal:
Answer
  • platelet aggregation within an atherosclerotic coronary artery.
  • decreased ventricular diastolic filling time.
  • embolisation of plaque from the aorta.
  • smooth muscle dysplasia in the coronary artery.

Question 22

Question
A 75-year-old male presents with severe chest pain. Lab tests in hospital reveal elevated levels of creatine kinase and lactic dehydrogenase. These elevated levels indicate probable:
Answer
  • Unstable angina
  • Hypertension
  • Stable angina
  • Myocardial infarction

Question 23

Question
A 60-year-old female had a myocardial infarction. She was brought to hospital 30 minutes later. She survived but now has impaired ventricular function because:
Answer
  • there was a temporary alteration in electrolyte balance.
  • the resulting ischaemia led to hypoxic injury and myocardial cell death.
  • there was too much stress on the heart.
  • the cells became hypertrophic.

Question 24

Question
Low-density lipoproteins (LDL):
Answer
  • transport cholesterol from cells to the liver for excretion.
  • contain only small amounts of cholesterol.
  • promote atheroma development.
  • are associated with low intake of saturated fats.

Question 25

Question
Which of the following patients will likely experience difficulty in maintaining lipoprotein synthesis resulting in elevated LDL levels?
Answer
  • A 35-year-old patient with a history of hepatitis C and B with end-stage liver disease
  • A 44-year-old female admitted for hysterectomy due to cervical cancer with metastasis
  • A 27-year-old patient with pancreatitis related to alcohol abuse
  • A 55-year-old male admitted for exacerbation of chronic obstructive pulmonary disease (COPD)

Question 26

Question
Hypertension can be classified into the following three major categories:
Answer
  • Active, passive, fatal
  • Primary, secondary, tertiary
  • Subacute, acute, chronic
  • Essential, secondary, malignant

Question 27

Question
Where do atherosclerotic plaques occur most often?
Answer
  • in large, abdominal veins
  • at points of arterial vessel bifurcation
  • in myocardial capillaries
  • all answers are correct

Question 28

Question
Calcium-channel blocking drugs are effective in:
Answer
  • decreasing all types of cardiac arrhythmias.
  • reducing the risk of blood clotting.
  • decreasing the attraction of cholesterol into lipid plaques.
  • reducing cardiac and smooth muscle contractions.

Question 29

Question
Which of the following confirms the presence of a myocardial infarction?
Answer
  • The presence of elevated serum cardiac enzymes and triglycerides
  • Serum cardiac enzymes markers released from necrotic cells and ECG changes
  • Leukocytosis and elevated C-reactive protein
  • A full description of the pain, including the sequence of development

Question 30

Question
High levels of which of the following lipoproteins can be beneficial?
Answer
  • IDL
  • LDL
  • HDL
  • VLDL

Question 31

Question
A 52-year-old male is diagnosed with primary hypertension. He could be treated with a drug that acts by which of the following mechanisms?
Answer
  • calcium channel agonist
  • diuretic
  • angiotensin ll agonist
  • beta-adrenergic agonist

Question 32

Question
Claudication describes, or is related to:
Answer
  • Painful cramps in skeletal muscles due to peripheral vascular disease
  • Painful upper limbs due to hypertension
  • Chest pain
  • All answers are correct

Question 33

Question
The basic pathophysiology of myocardial infarction is best described as:
Answer
  • total obstruction of a coronary artery, which causes myocardial necrosis.
  • irregular heart rate and force, reducing blood supply to coronary arteries.
  • temporary vasospasm that occurs in a coronary artery.
  • cardiac output that is insufficient to meet the needs of the heart and body.

Question 34

Question
Factors that may precipitate an angina attack include all of the following EXCEPT:
Answer
  • eating a large meal.
  • engaging in an angry argument.
  • running upstairs on a hot day.
  • taking a nap.

Question 35

Question
Typical early signs or symptoms of myocardial infarction include:
Answer
  • persistent chest pain radiating to the left arm, pallor, and rapid, weak pulse.
  • brief, substernal pain radiating to the right arm, with labored breathing.
  • bradycardia, increased blood pressure, and severe dyspnoea.
  • flushed face, rapid respirations, left-side weakness, and numbness.

Question 36

Question
Four patients were admitted to the emergency department with severe chest pain. All were given preliminary treatment with aspirin, morphine, and nitrates and were monitored by ECG. Which patient most likely experienced myocardial infarction?
Answer
  • A 67-year-old female whose pain started at 2 AM while she was asleep and responded to nitrates; the ECG showed arrhythmias and ST-segment elevation; cardiac markers remained stable.
  • An 80-year-old woman whose pain started at 6 AM shortly after awakening and was not relieved by nitrates or rest; the ECG showed ST-segment elevation; levels of cardiac markers subsequently rose.
  • A 33-year-old male whose pain started at 7 AM during moderate exercise and was relieved by nitrates; ECG was normal; cardiac markers remained stable.
  • A 61-year-old man whose pain started at 9 AM during a short walk and responded to nitrates, but not to rest; ECG and cardiac markers remained stable, but anginal pattern worsened.

Question 37

Question
Cigarette smoking is a risk factor in coronary artery disease because smoking:
Answer
  • promotes platelet adhesion.
  • increases serum HDL levels.
  • decreases serum lipid levels.
  • reduces vasoconstriction and peripheral resistance.

Question 38

Question
When comparing angina with myocardial infarction (MI), which statement is true?
Answer
  • Pain is more severe and lasts longer with angina than with MI.
  • Both angina and MI cause tissue necrosis.
  • Angina pain is relieved by rest and intake of nitroglycerin; the pain of MI is not.
  • Angina often occurs at rest; MI occurs during a stressful time.

Question 39

Question
Which of the following hypertensive individuals is most likely to have his or her high blood pressure diagnosed as secondary rather than essential?
Answer
  • A 40-year-old smoker who eats excessive amounts of salt and saturated fats
  • An African American man who leads a sedentary lifestyle
  • A 69-year-old woman who is an executive in a large corporation
  • A 51-year-old male who has been diagnosed with glomerulonephritis

Question 40

Question
A 54-year-old man with a long-standing diagnosis of essential hypertension is meeting with his physician. The patient's physician would anticipate that which of the following phenomena is most likely occurring?
Answer
  • The conversion of angiotensin I to angiotensin II in his lungs causes increases in blood pressure and sodium reabsorption.
  • The patient's juxtaglomerular cells are releasing aldosterone as a result of sympathetic stimulation.
  • ADH is exerting an effect on his chemoreceptors and baroreceptors resulting in vasoconstriction.
  • Adrenalin from his adrenal gland is initiating the renin -angiotensin- aldosterone system.

Question 41

Question
What condition refers to thrombotic events associated with venous wall inflammation, and occurs in any part of the body?
Answer
  • phlebothrombosis
  • deep vein thrombosis
  • varicose veins
  • thrombophlebitis

Question 42

Question
Which vessel/s is/are particularly prone to aneurysm?
Answer
  • aorta
  • superior vena cava
  • small peripheral arteries of the legs
  • inferior vena cava

Question 43

Question
A drug taken in small doses on a continuing basis to reduce platelet adhesion is:
Answer
  • Aspirin
  • Streptokinase
  • Heparin
  • Ibuprofen

Question 44

Question
A venous disorder in which a thrombus forms spontaneously in a vein without prior inflammation is referred to as:
Answer
  • Venous stenosis
  • Thrombophlebitis
  • Fibrothrombosis
  • Phlebothrombosis

Question 45

Question
A general term for all types of arterial changes, but most often applied to loss of elasticity of arteries and arterioles, is:
Answer
  • Vascular stenosis
  • Atherosclerosis
  • Arteriosclerosis
  • Osteoporosis

Question 46

Question
A modifiable factor that increases the risk for atherosclerosis is:
Answer
  • leading a sedentary lifestyle.
  • being female and older than 40 years of age.
  • familial hypercholesterolemia.
  • excluding saturated fats from the diet.

Question 47

Question
A coronary artery disease that occurs when the blood flow to the myocardial cells is interrupted for an extended period of time, followed by necrosis, is referred to as:
Answer
  • Angina pectoris
  • Atherosclerosis
  • Myocardial infarction
  • Dysrhythmias

Question 48

Question
Which antihypertensive drug group should be avoided when a person has a pre-existing respiratory condition like asthma?
Answer
  • Calcium channel antagonists.
  • Diuretics.
  • β-blockers.
  • ACE inhibitors.

Question 49

Question
Adverse effects such as an unproductive cough sometimes occur with which category of medicine frequently used for treatment of hypertension?
Answer
  • Calcium channel blockers
  • Diuretics
  • β-blockers
  • ACE inhibitors

Question 50

Question
Factors that may lead to the formation of varicose veins include:
Answer
  • Trauma, lying in bed for prolonged periods, and liver failure
  • Pregnancy, vitamin deficiencies, and mitral valve defects
  • Vein valve damage, wearing tight clothing, and crossing legs
  • Diet, exercise, and obesity

Question 51

Question
How do ACE inhibitors work?
Answer
  • They block Angiotensinogen production in the lungs
  • They block the enzyme cyclooxygenase
  • They block the formation of the enzyme renin by the kidney
  • They block the conversion of Angiotension 1 to Angiotensin 2

Question 52

Question
A 13-year-old boy has had a sore throat for at least a week and has been vomiting for 2 days. His glands are swollen, and he moves stiffly because his joints hurt. Throat cultures show infection with group A streptococci. This child is at high risk for?
Answer
  • mitral valve stenosis.
  • vasculitis.
  • myocarditis.
  • infective endocarditis.

Question 53

Question
A client has been diagnosed with mitral valve stenosis following his recovery from rheumatic fever. Which of the following teaching points would be most accurate to convey to the client?
Answer
  • Your heart's mitral valve isn't closing properly so blood is flowing backward in your heart and eventually into your lungs.
  • The valve between your left ventricle and left atria is infected and isn't allowing enough blood through.
  • The normal tissue that makes up the valve between the right sides of your heart has stiffened.
  • Your mitral valve isn't opening up enough for blood to flow into the part of your heart that sends blood into circulation.

Question 54

Question
On a routine physical exam visit, the physician mentions that he hears a new murmur. The patient gets worried and asks, “What does this mean?” The physician responds;
Answer
  • One of your heart valves is not opening properly. We need to do an echocardiogram to see which valve is having problem.
  • This may make you a little more fatigued than usual. Let me know if you start getting dizzy or light-headed.
  • This could be caused by an infection. Have you been feeling well the past few weeks?
  • It would be caused by stress. Let's keep our eye on it and see if it goes away with your next visit.

Question 55

Question
A 34-year-old man who is an intravenous drug user has presented to the emergency department with malaise, abdominal pain, and lethargy. The health care team wants to rule out endocarditis as a diagnosis. Staff of the department would most realistically anticipate which of the following sets of diagnostics?
Answer
  • CT of the heart, chest x-ray, and ECG
  • ECG, blood pressure, and stress test
  • Echocardiogram, blood cultures, and temperature
  • Cardiac catheterization, chest x-ray, electrolyte measurement, and white cell count

Question 56

Question
A physician has ordered the measurement of a cardiac patient's electrolyte levels as part of the client's morning blood work. Which of the following statements best captures the importance of potassium in the normal electrical function of the patient's heart?
Answer
  • Potassium catalyzes the metabolism of ATP, producing the gradient that results in electrical stimulation.
  • Potassium is central to establishing and maintaining the resting membrane potential of cardiac muscle cells.
  • The reciprocal movement of one potassium ion for one sodium ion across the cell membrane results in the production of an action potential.
  • The impermeability of cardiac cell membranes to potassium allows for action potentials achieved by the flow of sodium ions.

Question 57

Question
The initial medical management for a symptomatic patient with obstructive hypertrophic cardiomyopathy (HCM) would be administering a medication to block the effects of catecholamines. The nurse will anticipate administering which of the following medications?
Answer
  • Propranolol, a beta-adrenergic blocker
  • Lanoxin, an inotropic
  • Lisinopril, an ACE inhibitor
  • Lasix, a diuretic

Question 58

Question
A 63-year-old male client has been diagnosed with a bundle branch block. How will this client's care team most likely expect his condition to be expressed diagnostically?
Answer
  • His ECG will show a flattened P wave as a result of impaired atrial depolarization.
  • His ECG will show an inordinately wide QRS complex because impulses are bypassing the normal conduction tissue.
  • Conduction from the Purkinje fibers to the bundle branches is compromised by inadequate conduction.
  • His AV node will be performing the primary pacemaker role due to inadequacy of the SA node.

Question 59

Question
You are looking after a 61-year-old male client in the hospital who has presented with a new onset of atrial fibrillation. Which of the following courses of treatment would you most likely expect the attending physician to initiate?
Answer
  • Diuretics, total bed rest, and cardioversion if necessary
  • Anticoagulants and beta-blockers to control rate
  • Antihypertensives and constant cardiac monitoring in a high acuity unit
  • Immediate cardioversion followed by surgery to correct the atrial defect

Question 60

Question
The amount of blood pumped by one ventricle in one minute, is called the
Answer
  • stroke volume.
  • cardiac output.
  • ejection fraction.
  • end-diastolic volume.

Question 61

Question
If someone's heart has a stroke volume of 70 ml and a heart rate of 90 beat/minute, the cardiac output would be
Answer
  • 70 ml/min
  • 6.3 L/min
  • 1.28 ml/min
  • 0.77 L/min

Question 62

Question
Any mechanism that increases heart rate is said to have a positive ____ effect.
Answer
  • chronotropic
  • inotropic
  • cholinergic
  • feedback

Question 63

Question
Stroke volume is regulated by all of the following except
Answer
  • end-diastolic volume
  • peripheral resistance
  • cardiac output
  • contractility

Question 64

Question
The preload acting on a ventricle is equivalent to that chamber's
Answer
  • end-diastolic volume
  • stroke volume
  • contractility
  • ejection fraction

Question 65

Question
The afterload imposed on a ventricle refers to
Answer
  • the amount of blood added to a ventricle by atrial systole
  • the total peripheral resistance opposing the ejection of blood
  • its end-systolic volume, the blood left after contraction is complete
  • the ejection fraction, or percentage of EDV ejected by ventricular systole

Question 66

Question
The Frank-Starling law of the heart describes the proportional relationship between
Answer
  • stroke volume and end-diastolic volume
  • stroke volume and cardiac output
  • stroke volume is and total peripheral resistance
  • the left and right ventricles

Question 67

Question
A positive inotropic agent is something that
Answer
  • decreases the contractility of myocardial fibers
  • increases the contractility of myocardial fibers
  • reduces the heart rate in positive feedback loop
  • increases the heart rate in positive feedback loop

Question 68

Question
A nurse is administering morning medications to a number of patients on a medical unit. Which of the following medication regimens is most suggestive that the patient has a diagnosis of heart failure?
Answer
  • Anticoagulant, antihypertensive, calcium supplement
  • Beta-blocker, potassium supplement, anticoagulant
  • Diuretic, ACE inhibitor, beta-blocker
  • Antihypertensive, diuretic, antiplatelet aggregator

Question 69

Question
A nurse will be providing care for a female patient who has a diagnosis of heart failure that has been characterized as being primarily right sided. Which of the following statements best describes the presentation that the nurse should anticipate? The client
Answer
  • has cyanotic lips and extremities, low urine output, and low blood pressure.
  • has pitting edema to the ankles and feet bilaterally, decreased activity tolerance, and occasional upper right quadrant pain.
  • complains of dyspnoea and has adventitious breath sounds on auscultation (listening).
  • has a distended bladder, facial edema, and nighttime difficulty breathing.

Question 70

Question
A 3-year-old child with right-sided heart failure has been admitted for worsening of his condition. Which of the following assessments would be considered one of the earliest signs of systemic venous congestion in this toddler?
Answer
  • Breathlessness with activity
  • Increased urine output
  • Enlargement of the liver
  • Excessive crying

Question 71

Question
The nurse working in the ICU knows that chronic elevation of left ventricular end-diastolic pressure will result in the patient displaying which of the following clinical manifestations?
Answer
  • Chest pain and intermittent ventricular tachycardia
  • Dyspnoea and crackles in bilateral lung bases
  • Petechia and spontaneous bleeding
  • Muscle cramping and cyanosis in the feet

Question 72

Question
Which one of the following is not a pathophysiological change associated with heart failure?
Answer
  • Decreased angiotensin II production.
  • Decreased myocardial contractility.
  • Increased myocardial oxygen demand.
  • Cardiac remodelling.

Question 73

Question
Pulmonary symptoms, such as dyspnoea and cough, common to left heart failure are a result of:
Answer
  • pulmonary vascular congestion.
  • hypoxaemia.
  • inflammatory pulmonary oedema.
  • bronchoconstriction.

Question 74

Question
Adverse effects such as an unproductive cough and taste disturbances may occur from which category of medicine frequently used for treatment of heart failure?
Answer
  • Aldosterone antagonists.
  • Cardiac glycosides.
  • ACE inhibitors.
  • Diuretics.

Question 75

Question
Right heart failure will likely cause:
Answer
  • non-pitting oedema in the arms, resulting from decreased arterial pressure
  • pitting oedema in the lower legs, resulting from increased venous pressure
  • pulmonary oedema from increased left ventricular filling
  • all answers are correct

Question 76

Question
A 65-year-old male is diagnosed with chronic pulmonary disease and elevated pulmonary vascular resistance. Which of the following heart failures generally results from this condition?
Answer
  • low-output failure
  • right heart failure
  • high-output failure
  • left heart failure

Question 77

Question
Excess preload can be reduced by:
Answer
  • use of antidiuretics
  • use of diuretics
  • increasing volume intake, that is, drinking more fluid
  • all answers are correct

Question 78

Question
In terms of cardiac pathology, the greatest danger of untreated rheumatic fever is:
Answer
  • myocardial hypertrophy
  • damage to heart valves
  • atherosclerosis
  • acute right-side heart failure

Question 79

Question
An incompetent mitral valve would cause:
Answer
  • decreased pressure in the left atrium.
  • decreased output from the left ventricle.
  • hypertrophy of the right ventricle.
  • increased blood to remain in the right atrium.

Question 80

Question
A very rapid heart rate reduces cardiac output because:
Answer
  • conduction through the AV node is impaired.
  • ventricular filling is reduced.
  • venous return is increased.
  • ventricular fibrillation develops immediately.

Question 81

Question
Pericarditis causes a reduction in cardiac output as a result of which of the following?
Answer
  • Delays in the conduction system, interfering with cardiac rhythm
  • Excess fluid in the pericardial cavity, which decreases ventricular filling
  • Weak myocardial contractions due to friction rub
  • Incompetent valves, which allow regurgitation of blood

Question 82

Question
A patient, who is experiencing some angina associated with tachycardia, has been placed on verapamil, a calcium channel blocker. Knowing the mechanism of action of this medication, you should assess this patient for which of the following adverse reactions?
Answer
  • Ventricular tachycardia
  • Bradycardia
  • Increased cardiac output
  • Sudden asystole

Question 83

Question
Which of the following drugs improves cardiac efficiency by slowing the heart rate and increasing the force of cardiac contractions?
Answer
  • Epinephrine
  • Nifedipine
  • Digoxin
  • Furosemide

Question 84

Question
Heart block, in which a conduction delay at the AV node results in intermittent missed ventricular contractions, is called:
Answer
  • total heart block.
  • first-degree block.
  • second-degree block.
  • bundle-branch block.

Question 85

Question
Atrial fibrillation may:
Answer
  • increase the risk of thromboembolism
  • induce an irregular heart rate
  • increase turbulence within the atrial chamber
  • all answers are correct

Question 86

Question
The current optimal drug therapy in the management of heart failure is:
Answer
  • a non-selective β-blocker.
  • the dopamine agonist, dobutamine.
  • the cardiac glycoside, digoxin.
  • an ACE inhibitor and a diuretic.

Question 87

Question
Which one of the following would you not expect to be administered to a patient suffering heart failure?
Answer
  • diuretic
  • digoxin
  • beta-agonist
  • ACE inhibitor

Question 88

Question
A 67-year-old female has chronic gastrointestinal bleeding. The primary cause of her anaemia is:
Answer
  • bone marrow failure
  • folate deficiency
  • vitamin B12 deficiency
  • iron deficiency

Question 89

Question
Maternal-fetal blood incompatibility may exist in which of the following conditions?
Answer
  • The mother is Rh-negative and the fetus is Rh-positive.
  • The mother has type AB blood and the fetus has type B blood.
  • The mother has type A blood and the fetus has type O blood.
  • The mother is Rh-positive and the fetus is Rh-negative.

Question 90

Question
A 5-year-old male is diagnosed with normocytic-normochromic anaemia. Which of the following anaemias falls into this category?
Answer
  • thalassaemia
  • haemolytic anaemia
  • pernicious anaemia
  • iron deficiency anaemia

Question 91

Question
A 35-year-old female is diagnosed with vitamin B12 deficiency anaemia. A decrease in which of the following is the most likely cause?
Answer
  • transferin
  • intrinsic factor
  • gastric enzymes
  • ferritin

Question 92

Question
Erythrocytes that are _____ contain an abnormally low concentration of haemoglobin.
Answer
  • macrocytic
  • hypochromic
  • microcytic
  • hyperchromic

Question 93

Question
Which of the following is typically not associated with anaemia?
Answer
  • increased haemolysis
  • lack of intrinsic factor
  • high dietary iron intake
  • disruption of haemoglobin chains

Question 94

Question
Those who live at high altitudes, or who receive extra erythropoietin, are likely to suffer:
Answer
  • primary polycythaemia
  • haemolytic disease
  • secondary polycythaemia
  • sickle cell anaemia

Question 95

Question
Anaemia due to inherited mutations that reduce the production of either alpha or beta haemoglobin chains is known as:
Answer
  • haemolytic anaemia
  • thalassaemia
  • pernicious anaemia
  • all answers are correct

Question 96

Question
Which blood disorder results from an autoimmune attack on parietal cells of the stomach wall?
Answer
  • haemolytic disease of the newborn
  • pernicious anaemia
  • microcytic anaemia
  • thalassaemia

Question 97

Question
What medical term is used to describe a marked reduction in platelets?
Answer
  • haemorrhoids
  • haemophilia
  • thrombocytopenia
  • polycythaemia

Question 98

Question
Multiple opportunistic infections develop with acute leukemia primarily because:
Answer
  • many circulating leukocytes are immature
  • severe anemia interferes with the immune response
  • the number of white blood cells is decreased
  • decreased appetite and nutritional intake reduce natural defences

Question 99

Question
The cause of leukaemia is unknown, but risk factors include ...
Answer
  • Exposure to high amounts of radiation
  • Genetic disorders such as Down's syndrome
  • Exposure to carcinogens such as benzene
  • All answers are correct

Question 100

Question
Why is excessive bleeding a common occurrence with acute leukaemia?
Answer
  • Deficit of calcium ions
  • Dysfunctional thrombocytes
  • Impaired production of prothrombin and fibrinogen
  • Decreased platelets

Question 101

Question
A 5-year-old female is diagnosed with acute leukaemia. She will most likely be treated with:
Answer
  • bone marrow transplant
  • immunotherapy
  • localised radiation therapy
  • chemotherapy

Question 102

Question
The Reed-Sternberg cell is diagnostic for:
Answer
  • chronic myeloblastic leukemia.
  • multiple myeloma
  • non-Hodgkin’s lymphoma
  • Hodgkin’s lymphoma

Question 103

Question
Which of the following applies to the leukaemia’s?
Answer
  • Exposure to chemicals is not considered a predisposing factor.
  • Lymphoid tissue produces abnormal leukocytes.
  • Chronic leukemias are more common in older people.
  • All answers are correct.

Question 104

Question
Predisposing factors to leukemia commonly include:
Answer
  • exposure to radiation.
  • certain fungal and protozoal infections.
  • familial tendency.
  • cigarette smoking.

Question 105

Question
What is the primary treatment for the leukemias?
Answer
  • Chemotherapy
  • Radiation
  • Surgery
  • Immunotherapy

Question 106

Question
Multiple myeloma is a malignant tumor involving:
Answer
  • bone cells.
  • lymph nodes.
  • plasma cells.
  • granulocytes.

Question 107

Question
In cases of polycythemia vera, blood pressure is most likely elevated as a result of:
Answer
  • increased renin and aldosterone secretions.
  • increased blood volume/viscosity
  • congested spleen and bone marrow.
  • frequent infarcts in the coronary circulation

Question 108

Question
Vitamin K is required by the liver to synthesize:
Answer
  • bilirubin
  • amino acids
  • prothrombin
  • heparin

Question 109

Question
_____________________ would result from a reduced number of RBCs in the blood?
Answer
  • Decreased haematocrit
  • Increased haemoglobin count
  • Decreased osmotic pressure of the blood
  • Increased risk of haemostasis

Question 110

Question
Haemolytic disease of the newborn is due to the presence of ________ and ________.
Answer
  • Rh+ foetal blood, Anti-Rh antibodies in maternal blood
  • Rh+ foetal blood, Rh+ maternal blood
  • Rh- foetal blood, Rh+ maternal blood
  • Anti-Rh antibodies in foetal blood, lack of antibodies in maternal blood

Question 111

Question
Lymphomas describe white blood cell malignancies that:
Answer
  • do not involve bone marrow tissue initially
  • cause marked proliferation of white blood cells
  • initiate as cancers of lymphoid cells
  • all answers are correct

Question 112

Question
Malignant neoplasms involving lymphocyte proliferation in the lymph nodes are called:
Answer
  • lymphomas
  • leukaemias
  • myelomas
  • lymphocytomas

Question 113

Question
Which of the following substances acts as an anticoagulant?
Answer
  • Vitamin K
  • Prothrombin
  • Heparin
  • Fibrinogen

Question 114

Question
A 67-year-old female is admitted to the ED with a diagnosis of polycythaemia vera. This has occurred as a result of:
Answer
  • dehydration.
  • blood doping.
  • exposure to high altitude.
  • abnormal proliferation of red cells.

Question 115

Question
In disseminated intravascular coagulation (DIC), active bleeding occurs after intravascular clotting because:
Answer
  • clotting factors are depleted.
  • tissue factor (TF) is inactivated.
  • prothrombin is activated.
  • inflammatory mediators are released.

Question 116

Question
________ describes a condition where malignant white cells produce vast quantities of abnormal immunoglobulins that ultimately destroy bone.
Answer
  • Multiple myeloma
  • Chronic lymphocytic leukaemia
  • Agranulocytosis
  • Non-Hodgkin lymphoma

Question 117

Question
A neoplastic disorder that involves malignant plasma cells that replace the bone marrow and erodes bone is referred to as:
Answer
  • Aplastic anemia
  • Multiple myeloma
  • Leukaemia
  • Non-Hodgkin lymphoma

Question 118

Question
What are the typical early clinical manifestations of anaemia?
Answer
  • Jaundice, cyanosis
  • Pallor, dyspnea, and fatigue
  • Chest pain, palpitations
  • Bradycardia, cyanosis

Question 119

Question
Which of the following individuals is at highest risk for developing a vitamin B12 deficiency anaemia?
Answer
  • a 47-year-old male who had a gastrectomy procedure (removal of the stomach)
  • a 3-year-old female who is a fussy eater
  • a 64-year-old male with a history of duodenal ulcers and gastrointestinal bleeding
  • a 26-year-old female in the second trimester of her first pregnancy

Question 120

Question
A diverse group of neoplasms developing from the proliferation of malignant lymphocytes in the lymphoid system is referred to as:
Answer
  • Leukaemias
  • Microcytic anaemias
  • Lymphomas
  • Lymphatic anaemias

Question 121

Question
In which of the following individuals would a clinician most suspect multiple myeloma as a diagnosis?
Answer
  • A 70-year-old woman whose blood work reveals large numbers of immature granulocytes
  • An 81-year-old male resident of a long-term care home who has an uncommon bacterial pneumonia and who is unable to produce a fever
  • A 40-year-old man who has had three broken bones over the past 6 months and whose serum calcium and creatinine levels are elevated
  • A 68-year-old former coal miner who has white cell levels exponentially higher than normal ranges

Question 122

Question
A 34-year-old male presents in the ED with extreme fatigue and shortness of breath. His skin and sclera appear to have a yellowish discoloration. These findings are consistent with which type of anaemia?
Answer
  • pernicious anaemia
  • aplastic anaemia
  • iron deficiency anaemia
  • haemolytic anaemia

Question 123

Question
A 40-year-old female develops disseminated intravascular coagulation (DIC). The most likely cause of this condition is:
Answer
  • lack of vitamin B12.
  • blood transfusion.
  • snake venom.
  • sepsis.

Question 124

Question
A 52-year-old male IV drug user 5years ago was diagnosed with hepatitis C. He is now experiencing impaired blood clotting. A decrease in which of the following vitamins is most likely the cause?
Answer
  • E
  • K
  • B12
  • D

Question 125

Question
Chronic blood loss causes anaemia because of the:
Answer
  • lower metabolic rate
  • shortened life span of the erythrocytes
  • loss of protein and electrolytes
  • smaller amount of recycled iron available

Question 126

Question
What is the basic abnormality in thalassemia?
Answer
  • Several amino acids in the Haem chains have been replaced by substitute amino acids.
  • The iron molecule is displaced in haemoglobin.
  • More than four globin chains are found in the erythrocytes.
  • There is failure to synthesize either the alpha or beta chains in the haemoglobin molecule.

Question 127

Question
A 60-year-old woman is suspected of having non-Hodgkin lymphoma (NHL). Which of the following aspects of her condition would help to rule out Hodgkin lymphoma?
Answer
  • The woman complains of recent debilitating fatigue.
  • Her neoplasm originates in secondary lymphoid structures.
  • The lymph nodes involved are located in a large number of locations in the lymphatic system.
  • The presence of Reed-Sternberg cells has been confirmed.

Question 128

Question
One of the reasons non-Hodgkin’s lymphomas are harder to treat than Hodgkin’s lymphomas is that they
Answer
  • are not affected by the newer drug treatments.
  • are asymptomatic until they reach stage IV.
  • tend to be much larger than Hodgkin’s lymphomas.
  • involve multiple nodes and widespread metastases.

Question 129

Question
Individuals with liver disease often suffer from coagulation disorders because:
Answer
  • treatment medications for liver failure cause fibrinolysis.
  • the liver is often the site of platelet pooling.
  • high levels of bilirubin interfere with the clotting system.
  • clotting factors are produced in the liver.

Question 130

Question
In individuals with acute leukaemia, the increased number of malignant leukocytes leads to: 1. decreased haemoglobin. 2. thrombocytopenia. 3. bone pain only with increased activity. 4. splenomegaly.
Answer
  • 1,3
  • 1,2,4
  • 1,2,3,4
  • 2,3,4
Show full summary Hide full summary

Similar

Application of technology in learning
Jeff Wall
OCR Biology AS level (f211) flashcards/revision notes
Dariush Zarrabi
Girls' and Boys'Education - Catherine and Kaitlyn
catherine.dannib
A level Henry VIII: Foreign policy
avocadolover
English Language Terms
ekimlauretta
Biology F212 - Biological molecules 1
scarlettcain97
Creative Writing
amberbob27
PSBD TEST # 3
yog thapa
Relationships in A Streetcar Named Desire
Emily Garvin-Howard
Final Exam
Ms. Wong-Lee